Imagination is more important than knowledge... Albert Einstein

                                                                                   Guess is more important than calculation --- Knowhowacademy.com

 

solid geometry - Angle

For COMPETITION
Number of Total Problems: 14.
FOR PRINT ::: (Book)

Problem Num : 1
From : NCTM
Type: None
Section:solid geometry 
Theme:None
Adjustment# :
Difficulty: 1

Category Angle
Analysis

Solution/Answer


Problem Num : 2
From : NCTM
Type: None
Section:solid geometry 
Theme:None
Adjustment# :
Difficulty: 1

Category Angle
Analysis

Solution/Answer


Problem Num : 3
From : NCTM
Type: None
Section:solid geometry 
Theme:None
Adjustment# :
Difficulty: 1

Category Angle
Analysis

Solution/Answer


Problem Num : 4
From : NCTM
Type: None
Section:solid geometry 
Theme:None
Adjustment# :
Difficulty: 1

Category Angle
Analysis

Solution/Answer


Problem Num : 5
From : NCTM
Type: None
Section:solid geometry 
Theme:None
Adjustment# :
Difficulty: 2

Category Angle
Analysis

Solution/Answer


Problem Num : 6
From : AMC8
Type:
Section:solid geometry 
Theme:
Adjustment# : 0
Difficulty: 1
'

The degree measure of angle A is

unitsize(12);draw((0,0)--(20,0)--(1,-10)--(9,5)--(18,-8)--cycle);draw(arc((1,-10),(1+19/sqrt(461),-10+10/sqrt(461)),(25/17,-1...

	ext{(A)} 20 qquad 	ext{(B)} 30 qquad 	ext{(C)} 35 qquad 	ext{(D)} 40 qquad 	ext{(E)} 45

'
Category Angle
Analysis

Solution/Answer

Angle-chasing using the small triangles:

Use the line below and to the left of the 110^circ angle to find that the rightmost angle in the small lower-left triangle is 180 - 110 = 70^circ.

Then use the small lower-left triangle to find that the remaining angle in that triangle is 180 - 70 - 40 = 70^circ.

Use congruent vertical angles to find that the lower angle in the smallest triangle containing A is also 70^circ.

Next, use line segment AB to find that the other angle in the smallest triangle contianing A is 180 - 100 = 80^circ.

The small triangle containing A has a 70^circ angle and an 80^circ angle. The remaining angle must be 180 - 70 - 80 = oxed{30^circ, B}

Answer:



Problem Num : 7
From : AMC12
Type:
Section:solid geometry 
Theme:
Adjustment# : 0
Difficulty: 1
'

Find the degree measure of an angle whose complement is 25% of its supplement.

mathrm{(A)  48 } qquad mathrm{(B)  60 } qquad mathrm{(C)  75 } qquad mathrm{(D)  120 } qquad mathrm{(E)  150 }


'
Category Angle
Analysis

Solution/Answer

4(90-x)=(180-x)

360-4x=180-x

3x=180

x=60 Rightarrow mathrm {(B)}

Answer:



Problem Num : 8
From : AMC10B
Type:
Section:solid geometry 
Theme:
Adjustment# : 0
Difficulty: 1
'

The angles of quadrilateral ABCD satisfy angle A=2 angle B=3 angle C=4 angle D. What is the degree measure of angle A, rounded to the nearest whole number?

	extbf{(A) } 125 qquad	extbf{(B) } 144 qquad	extbf{(C) } 153 qquad	extbf{(D) } 173 qquad	extbf{(E) } 180

'
Category Angle
Analysis

Solution/Answer

The sum of the interior angles of any quadrilateral is 360^circ. egin{align*}360 &= angle A + angle B + angle C + angle D\&= angle A + frac{1}{2}A + frac{1}{3}A + frac{1}{... angle A = 360 cdot frac{12}{25} = 172.8 approx oxed{mathrm{(D)  } 173}

Answer:



Problem Num : 9
From : AMC10
Type:
Section:solid geometry 
Theme:
Adjustment# : 0
Difficulty: 1
'

Triangles ABC and ADC are isosceles with AB=BC and AD=DC. Point D is inside triangle ABC, angle ABC measures 40 degrees, and angle ADC measures 140 degrees. What is the degree measure of angle BAD?

mathrm{(A)} 20qquad mathrm{(B)} 30qquad mathrm{(C)} 40qquad mathrm{(D)} 50qquad mathrm{(E)} 60

'
Category Angle
Analysis

Solution/Answer

2007 AMC12A-6.png

We angle chase, and find out that:

  • DAC=frac{180-140}{2} = 20
  • BAC=frac{180-40}{2} = 70
  • BAD=BAC-DAC=50 mathrm{(D)}

Answer:



Problem Num : 10
From : AMC10
Type:
Section:solid geometry 
Theme:
Adjustment# : 0
Difficulty: 1
'

Suppose that frac{2x}{3}-frac{x}{6} is an integer. Which of the following statements must be true about x?

mathrm{(A)} 	ext{It is negative.}\qquadmathrm{(B)} 	ext{It is even, but not necessarily a multiple of 3.}\qquadmat...

'
Category Angle
Analysis

Solution/Answer

frac{2x}{3}-frac{x}{6}quadLongrightarrowquadfrac{4x}{6}-frac{x}{6}quadLongrightarrowquadfrac{3x}{6}quadLongright... For frac{x}{2} to be an integer, x must be even, but not necessarily divisible by 3. Thus, the answer is mathrm{(B)}.

Answer:



Array ( [0] => 3718 [1] => 3721 [2] => 3722 [3] => 3752 [4] => 3775 [5] => 8265 [6] => 8793 [7] => 8078 [8] => 7850 [9] => 7869 ) 141  2